Find the slope m and y-intercept of the equation:
y = (-1/5)x + 7

Answers

Answer 1
Answer:

m = -1/5

b = 7

Step-by-step explanation:

The equation is written is slope-intercept form where m = slope and b = y-intercept (y = mx + b)

Thus, the slope is -1/5 and the y-intercept is 7

Related Questions

Mua hàng hóa 10000kg về nhập kho,Đơn giá 200 000đ/kg,thuế gtgt là 10%,trả bằng chuyển khoản 50%,còn nợ người bán.Chi phí vận chuyển 2 100 000 bao gồm thueest gtgt 5% trả tiền mặt

Answers

I don’t think so it’s right question

Find the missing side lengths.

Answers

Answer:

x is 18, and y is 9 √ 3

Step-by-step explanation:

Since this is a right triangle that contains a 30 degree angle, we know this is a special right triangle, and the missing angle is 60 degrees since a triangle is 180 degrees, and 180 - 90 - 30 = 60.

The relationship is listed below:

The side opposite of the 30 degrees can be represented as a variable, say "p", and the hypotenuse which is x in your question is twice this. The side opposite of the 60 degree angle is x + √3

So x is 18, and y is 9√3

pLEASE help best and right answer gets brainliest

Answers

1) is 9
2) is 2

Absolute value of a number is the distance a number is from 0.

Step-by-step explanation:

| - 5 | + | - 4 |

5 + 4

= 9

| - 6| - 4

6 - 4

2

I hope this answers your question.

What is the equation of the line that passes through the point (8,−3) and has an undefined slope?
HELP FAST PLS!!

Answers

Answer:

x = 8

Step-by-step explanation:

undefined slope => m = oo

passes (8, -3) => (x1, y1)

the equation is : y-y1 =m(x-x1)

y+3 = oo(x-8)

=> x -8 = (y+3)/oo

x -8 = 0

x = 8

so, the equation is: x = 8

Write the number in standard form as a decimal

Answers

Answer:

4.00810.1

Step-by-step explanation:

I hope it will help youplease make me brainlest

THANK U

Taylor wants to find the perimeter of a rectangular playground. The lenght of the playground measures (3x-20) metres. The width of the playground measures (2x+4) metres. What is the perimeter of the playground? ​

Answers

Answer:

Step-by-step explanation:

P = 2(3x-20) + 2(2x+4) = (6x-40) + (4x+8) = 10x-32

The required perimeter of the playground is 10x-32.

The length of the playground measures (3x-20) metres.
The width of the playground measures (2x+4) metres.

What is the perimeter?

Perimeter, is the measure of the figure on its circumference.

The Required perimeter is for the playground is given by
= 2(3x-20) + 2(2x+4)

= 10x-32

Thus the required perimeter of the playground is 10x-32.

learn more about perimeter here:

brainly.com/question/6465134

#SPJ2

Help with this Area question

Answers

Step 1: Find the area of the rectangle

A = base x height

A = 39 x 20

A = 780

Step 2: Find the area of the semi-circles

---Two semi-circles is the same as one whole circle, so I will be finding the area of one whole circle.

A = pi x r^2

A = pi x 10^2

A = 100pi = 314

Step 3: Find the area of the figure

Area = area of the rectangle - area of the semi-circles

A = 780 - 314

A = 466 cm^2

Hope this helps!

Answer:

466 cm^2

Step-by-step explanation:

This one is done basically the same as the other.

Rectangle = 20 x 39

Circle = (3.14) x 10^2

Rectangle = 780

Circle = 314

rectangle - circle

780 - 314 = 466

Which expression corresponds to this graph?

Answers

Answer:

C

Step-by-step explanation:

The expression is x>55 and 55 isn't included

Answer:

c

Step-by-step explanation:

Find the measure of the missing angles.

Answers

Answer:

Step-by-step explanation:

I need help answering this ASAP

Answers

Answer:

"D"

if you multiply by Conjugate

the denominator would end up A^2 - b^2

the answer has 25 - 10x

that is D

Step-by-step explanation:

Please help! Question and answers are in the pic

Answers

So far she worked 4 days at 5 1/2 hours a day for a total of 22 hours.

22 hours x $8.50 = $187

Subtract that from the cost of the computer:

899-187 = $712

She needs $712 more.

Amount she makes per shift: $8.50 x 5 1/2 hours = $46.75

Divide what she needs by amount per shift:

712 / 46.75 = 15.22 shifts

She needs to work 16 more shifts.

Find f such that f'(x) = 8x – 3. f(4) = 0​

Answers

Answer: y=29 / (4,29)

Step-by-step explanation:

By graphing [tex]f(x)=8x-3[/tex] and [tex]f(4)=0[/tex] on Desmos. You'll be able to find that when x is 4, y is 29.

Similarly, you can plugin 4 into the original equation ([tex]f(x)=8x-3[/tex]) Which looks like:

[tex]8(4)-3\\32-3=29[/tex]

Additionally, you can change [tex]f(x)=[/tex] to [tex]y=[/tex] as it is the exact same thing. With that in mind, you can do the same with [tex]f(x)=0[/tex] and just change it to x=4. As you're wanting to know what the y-value is when x=4.

Answer:

4x²-3x+c is our original equation

Step-by-step explanation:

we have an independent number I called this number c

put 4 from x and try to find c

f(4)=4*(4²)-3*(4)+c=0

we have to be careful about f(4) is 0

64-12+c=0 and c is -52 so our original equation is 4x²-3x-52

During a particularly dry growing season in a southern state, farmers noticed that there is a delicate balance between the number of seeds that are planted per square foot and the yield of the crop in pounds per square foot. The yields were the smallest when the number of seeds per square foot was either very small or very large.

What is the explanatory variable for this relationship?

yield of the crop
location of the farm
precipitation for the growing season
number of seeds planted per square foot

I think it's (D).
number of seeds planted per sf

Answers

Answer:

The guy above me is correct

Step-by-step explanation:

2022

Answer:

number of seeds planted per square foot

Step-by-step explanation:

response is the yield explained by how many seeds are planted

A poll of 2,060 randomly selected adults showed that 89% of them own cell phones. The technology display below results from a test of the claim that 91% of adults own cell phones. Use the normal distribution as an approximation to the binomial distribution, and assume a 0.01 significance level to complete parts (a) through (e).
Test of p=0.91 vs p≠0.91
Sample X N Sample p 95% CI Z-Value p-Value
1 1833
2,060 0.889806 ( 0.872035 , 0.907577 ) ~ 3.20 0.001
a. Is the test two-tailed, left-tailed, or right-tailed?∙
Left-tailed test∙
Two-tailed test∙
Right tailed test
b. What is the test statistic?
The test statistic is _____ (Round to two decimal places as needed.)
c. What is the P-value?
The P-value is _____ (Round to three decimal places as needed.)
d. What is the null hypothesis and what do you conclude about it?
Identify the null hypothesis.
A. H0:p<0.91∙
B. H0:p≠0.91∙
C. H0:p>0.91∙
D. H0:p=0.91.

Answers

Answer:

Two tailed test

Test statistic = 3.20

Pvalue = 0.001

H1 : p ≠ 0.91

Step-by-step explanation:

Given :

Test of p=0.91 vs p≠0.91

The use if not equal to ≠ sign in the null means we have a tow tailed test, which means a difference exists in the proportion which could be lesser or greater than the stated population proportion.

The test statistic :

This is the Z value from the table given = 3.20

The Pvalue = 0.001

Since Pvalue < α ;Reject H0

∫[tex]\frac{x+2019}{x^{2}+9 }[/tex]

Answers

Split up the integral:

[tex]\displaystyle\int\frac{x+2019}{x^2+9}\,\mathrm dx = \int\frac{x}{x^2+9}\,\mathrm dx + \int\frac{2019}{x^2+9}\,\mathrm dx[/tex]

For the first integral, substitute y = x ² + 9 and dy = 2x dx. For the second integral, take x = 3 tan(z) and dx = 3 sec²(z) dz. Then you get

[tex]\displaystyle \int\frac x{x^2+9}\,\mathrm dx = \frac12\int{2x}{x^2+9}\,\mathrm dx \\\\ = \frac12\int\frac{\mathrm du}u \\\\ = \frac12\ln|u| + C \\\\ =\frac12\ln\left(x^2+9\right)[/tex]

and

[tex]\displaystyle \int\frac{2019}{x^2+9}\,\mathrm dx = 2019\int\frac{3\sec^2(z)}{(3\tan(z))^2+9}\,\mathrm dz \\\\ = 2019\int\frac{3\sec^2(z)}{9\tan^2(z)+9}\,\mathrm dz \\\\ = 673\int\frac{\sec^2(z)}{\tan^2(z)+1}\,\mathrm dz \\\\ = 673\int\frac{\sec^2(z)}{\sec^2(z)}\,\mathrm dz \\\\ = 673\int\mathrm dz \\\\ = 673z+C \\\\ = 673\arctan\left(\frac x3\right)+C[/tex]

Then

[tex]\displaystyle\int\frac{x+2019}{x^2+9}\,\mathrm dx = \boxed{\frac12\ln\left(x^2+9\right) + 673\arctan\left(\frac x3\right) + C}[/tex]

2. Solve the following:
a. When six is added to four times a number the result is 50. Find the number.
b. The sum of a number and nine is multiplied by -2 and the answer is -8. Find the
number.
c. The length of a rectangular map is 37,5 cm and the perimeter is 125 cm. Find the
width.​

Answers

Answer:

a = 11

b = -5

c = 25

Step-by-step explanation:

a.

6 + 4x = 50

50 - 6 = 44

4x = 44

x = 11

b.

(x+9) * -2 = -8

(x+9) = 4

x = -5

c.

2(37.5) + 2(x) = 125

75 + 2x = 125

125 - 75 = 2x

50 = 2x

x = 25

If my answer is incorrect, pls correct me!

If you like my answer and explanation, mark me as brainliest!

-Chetan K

Coronado reported the following information for the current year: Sales (44000 units) $880000, direct materials and direct labor $440000, other variable costs $44000, and fixed costs $360000. What is Coronado’s break-even point in units?

a) 32727.
b) 40000.
c) 60923.
d) 36000.

Answers

The answer i think is c not positive tho

How to write this sum 24,6 in expanded notation

Answers

Answer:

2.46×10

Step-by-step explanation:

24.6

2.46 ×10

.....................

Determine if f(x, y) = 10 − x^2 − y^2
is increasing or decreasing at (7, −3) if we
take y to be constant and let x vary. Also determine if f(x, y) is increasing at
(7, −3) if we take x to be constant and let y vary.

Answers

Answer:

Step-by-step explanation:

f(x,y)=10-x^2-y^2

To find derivative of z w.r.t. x is treat any other variable as a constant.

dz/dx=0-2x-0

dz/dx=-2x

Evaluating this at (7,-3) gives us dz/dx=-2(7)=-14.

Since this result is negative, it mrans as x increases z decreases.

f(x,y)=10-x^2-y^2

To find derivative of z w.r.t. y is treat any other variable as a constant.

dz/dx=0-0-2y

dz/dx=-2y

Evaluating this at (7,-3) gives us dz/dy=-2(-3)=6.

Since this result is positive it mrans as y increases z decreases.

w^2+2w-42=0
what is the width and the length

Answers

Answer:

answers in the explanation cz I'm too lazy to type :(

not entirely sure tho

Step-by-step explanation:

w²+2w-42=0

*quadratic formula*

w= -1+ square root 43 m

or w= -1- square root 43 m

then since the length is 2m more than w

add 2 to both answers

l= 1+ square root 43 m

l=1- square root 43 m

9514 1404 393

Answer:

width: 5.557 mlength: 7.557 m

Step-by-step explanation:

Given:

  a rectangular patio of width w meters, length w+2 meters, and area 42 m²

Find:

  width and length

Solution:

The area is ...

  A = LW

  42 = w(w +2)

  43 = w² +2w +1 . . . . . . add 1 to complete the square

  √43 = w+1

  w = √43 -1 ≈ 5.557 . . . meters

  l = w+2 = √43 +1 ≈ 7.557 . . . meters

The width and length of the patio are 5.557 m and 7.557 m, respectively.

Which among the given schemes offers a monthly instalment of less than Rs 5000. ?

a) Scheme A
b) Scheme B
c) Scheme C
d) Both Scheme A and Scheme B​

Answers

Heyy I think it’s d it’s my answer but if u don’t think u don’t need to answer the answer I gave to u

What is the value of the expression
below?
(7)3

Answers

Answer:

21

Step-by-step explanation:

Need the help thanks guys

Answers

Answer:

Option D is the correct answer.

Step-by-step explanation:

The equation of the function is in vertex form. Thus, we can analyze the equation to determine the x and y values of the vertex. We know that the template format of a vertex form equation is as follows:

f(x) = a(x – h)2 + k . The only constants we need are h and k, where 'h' is the x-value of our vertex and 'k' is the y-value of our vertex.

The value of 'k' can be found quite simply by looking at the equation: -9.

The value of 'h' is a little trickier, as we must take into account the 'subtract' sign of the template equation, meaning that the '+7' in the given equation actually means that we are subtracting negative seven. Thus, the value of 'h' is -7.

Thus, the vertex of this graph is (-7,-9).

This means that option D is correct.

Answer:

The vertex is at ( -7, -9)

Step-by-step explanation:

y = (x+7)^2 -9

This is written in vertex form

y = a( x-h)^2 +k where ( h,k) is the vertex

y = ( x - -7)^2 + -9

The vertex is at ( -7, -9)

The difference between two positive integers is 7 and the sum of their squares is 949. What are the numbers?

Answers

Answer:

25 and 18

Step-by-step explanation:

Let's say that the first number is x and the second one is y.

First, the difference between them is 7, so x-y=7

Next, the sum of their squares is 949, so x²+y² = 949

We have

x-y=7

x²+y²=949

One thing we can do to solve this problem is to solve for x in the first equation, plug that into the second equation, and go from there

Adding y to both sides in the first equation, we have

x = 7 + y

Plugging that into the second equation for x, we have

(7+y)²+ y² = 949

expand

(7+y)(7+y) + y² = 949

49 + y² + 7y + 7y + y² = 949

combine like terms

2y² +14y + 49 = 949

subtract 949 from both sides to put this in the form of a quadratic equation

2y² + 14y - 900 = 0

divide both sides by 2

y² + 7y - 450 = 0

To factor this, we want to find 2 numbers that add up to 7 and multiply to -450.

The factors of 450 are as follows:

1, 2, 3, 5, 6, 9, 10, 15, 18, 25, 30, 45, 50, 75, 90, 150, 225, and 450.

Note that we want to find two numbers with a difference of 7, as one will have to be negative for the multiplication to end up at -450. Two numbers that stand out are 18 and 25. To make them add up to 7, 18 can be negative. We therefore have

y² + 25y - 18y - 450 = 0

y(y+25) - 18(y+25) = 0

(y-18)(y+25) = 0

Solving for 0,

y-18 = 0

add 18 to both sides

y=18

y+25 = 0

subtract 25 from both sides

y= -25

As the question states "two positive integers", this means that y must be positive, so y = 18. We know x-y=7, so

x-18 = 7

add 18 to both sides to isolate x

x = 25

We have that 2a+1=1 and b-a=1. What is the value of b?

Answers

Answer:

b=1

Step-by-step explanation:

2a+1 =1

Subtract 1 from each side

2a+1-1 = 1-1

2a=0

a=0

Now find b from the second equation

b-a =1

b-0=1

b=1

Answer:

Equation: 2a+1=1

Subtract 1 from both sides: 2a=0

Divide by 2: a=0

Equation: b-a=1

Substitute: b-0=1

Combine: b=1

So, b=1.

Let me know if this helps.

What is the equation of a line that passes through the point (1,8) and is perpendicular to the line whose equation is y=x/2+3?

Answers

Answer:

m=1/2

y-8=1/2(x-1)

y-8=1/2x-1/2

multiply through by 2

2y-16=x-1

2y-16+1-x=0

2y-15-x=0

2y-x-15=0

I NEED HELP PLEASE !!!

Answers

Answer: Because [tex]\frac{\pi }{3} =\frac{180\°}{3} =60\°[/tex], therefore [tex]\frac{\pi }{3} =60\°[/tex].

find the sum 38+39+40+41...+114+115

Answers

It seems like you want to find the sum of 38 to 115:

[tex] \displaystyle \large{38 + 39 + 40 + 41 + ... + 114 + 115}[/tex]

If we notice, this is arithmetic series or the sum of arithmetic sequences.

To find the sum of the sequences, there are three types of formulas but I will demonstrate only one and the best for this problem.

[tex] \displaystyle \large{S_n = \frac{n(a_1+a_n) }{2} }[/tex]

This formula only applies to the sequences that have the common difference = 1.

Given that a1 = first term of sequence/series, n = number of terms and a_n = last term

We know the first term which is 38 and the last term is 115. The problem here is the number of sequences.

To find the n, you can use the following formula.

[tex] \displaystyle \large{n = (a_n - a_1) + 1}[/tex]

Substitute an = 115 and a1 = 38 in the formula of finding n.

[tex] \displaystyle \large{n = (115 - 38) + 1} \\ \displaystyle \large{n = (77) + 1} \\ \displaystyle \large{n = 78}[/tex]

Therefore the number of sequences is 78.

Then we substitute an = 115, a1 = 38 and n = 78 in the sum formula.

[tex] \displaystyle \large{S_{78} = \frac{78(38+115) }{2} } \\ \displaystyle \large{S_{78} = \frac{39(38+115) }{1} } \\ \displaystyle \large{S_{78} = 39(153) } \\ \displaystyle \large \boxed{S_{78} = 5967}[/tex]

Hence, the sum is 5967.

Use the vertex
(h, k)
and a point on the graph
(x, y)
to find the general form of the equation of the quadratic function.
(h, k) = (−4, −1), (x, y) = (−7, 8)

Answers

Answer:

Step-by-step explanation:

Let the quadratic function be

y=a(x+4)²-1

∵(-7,8) lies on it.

8=a(-7+4)²-1

8+1=a(-3)²

9a=9

a=9/9=1

so quadratic function  is

f(x)=1(x+4)²-1

or

f(x)=x²+8x+16-1

so quadratic function is

f(x)=x²+8x+15

A general wishes to draw up his 7500 soldiers in the form of a square. After arranging, he found out that some of them are left out. How many soldiers were left out ?​

Answers

If you want to form a square of 7500 soldiers, the side of the square must be [tex]\sqrt{7500}\approx86.6[/tex] soliders.

But since you cannot have 0.6 solider, the general needs to find the closest perfect square to the number 7500 which is less than 7500.

That number is 7396 which when square rooted gives 86 soliders on the side.

Subtract 7396 from 7500 and get how many soliders were left out,

[tex]7500-7396=\boxed{104}[/tex]

Hope this helps :)

Other Questions
Fred is working on a new storage system for his organization. He needs it to provide storage for the entire private cloud in the data center. Which of the following is he most likely to deploy?a. SANb. NASc. NSAd. DAS What do you mean by photosynthesis?bore D D D D A 59.0 kg bungee jumper jumps off a bridge and undergoes simple harmonic motion. If the period of oscillation is 0.250 mins, what is the spring constant (in N/m) of the bungee cord, assuming it has negligible mass compared to that of the jumper During respiration, carbon dioxide diffuses from the blood across the alveolus and into the lungs. Which factor increases the diffusion rate of carbon dioxide? solve the equation on the interval [0,2pi] 5sec(x)+7=-3 Why is salvation not a license to sin? A sample of 34 observations is selected from a normal population. The sample mean is 15, and the population standard deviation is 3. Conduct the following test of hypothesis using the 0.10 significance level. H0: 14 H1: > 14 Required:a. Compute the value of the test statistic. b. What is the p-value? Please help me out . Find x please Rewrite the following without an exponent 3^-3 I need e and f And some explanation A fast moving vehicle travelling at a speed of 25.4 m/s comes up behind another vehicle which istravelling at a slower constant speed of 13.6 m/s. If the faster vehicle does not begin braking until itis 11.4 meters away from the car in front of it, what is the minimum acceleration that the faster carmust exhibit if it is to avoid colliding with the car in front? Assume that both cars are travelling in thepositive direction What is the slope of the line given by the equation y=11.2x? How far did east did Muslim territory spread during the period of the RightlyGuided Caliphs? Your readings for this unit discussed a research study in which participants were exposed to a live cold virus. What are your thoughts about this study? Would you be willing to participate in such research? Why or why not? I'm very bad at this o-o What are Three effects of 9/11 and 2 examples of each effect. according to who,is the definition of health Triangle ABC is the image of ABC, under a rotation about point Q. Determine the angle of rotation. According to the DIS section on socialization, research illustrates that children are moving away from traditional methods of interaction. This is due to an increased involvement in: multiply 4111 base 5 with 31 base 5